Paper 1 Flashcards

1
Q

Barry should not put in any evidence about what occurred at the mediation apart from the signed settlement agreement and evidence that this was concluded at the mediation at a particular date and time.
[B] Barry should put in evidence his own recollection of what happened at the mediation to show that he was not aware of any error that Dean believes that he made in relation to the strength of his case.
[C] Barry should put in evidence details of the negotiation between the parties to show that the settlement, if anything, was generous to Dean.
[D] Barry should put in evidence from both the mediator and Dean’s solicitor to show that Dean was not misled about the strength of his case.

A

[A] Barry should not put in any evidence about what occurred at the mediation apart from the signed settlement agreement and evidence that this was concluded at the mediation at a particular date and time.

[A] This is the correct answer. What occurred at a mediation is confidential save for the resulting agreement and the fact that the mediation took place (Jackson ADR Handbook (Third Edition, 2021) at 13.47, 13.48, 13.49, 13.52, 13.54). While there are some exceptions to confidentiality (Jackson ADR Handbook (Third Edition, 2021)at13.54, 13,55), none of those exceptions arise on the facts here. See also CPR 31.3, SA14.

How well did you know this?
1
Not at all
2
3
4
5
Perfectly
2
Q

[A] Asad is entitled to the costs of the proceedings up until 1 March 2024 but should pay RL’s costs and interest on those costs from that date, unless the court considers it unjust to make such an order.
[B] Asad is entitled to costs up until 19 December 2023 and interest on those costs, unless the court considers it unjust to make such an order.
[C] Asad is entitled to costs up until 19 December 2023 and RL is entitled to costs from that date, and interest on those costs, unless the court considers it unjust to make such an order.
[D] An order that there be no order for costs as Asad acted unreasonably in failing to accept the first or the second Part 36 offer, neither of which were beaten at trial.

A

Answer - [C] Asad is entitled to costs up until 19 December 2023 and RL is entitled to costs from that date, and interest on those costs, unless the court considers it unjust to make such an order.
[A] is not the best answer as costs shifting occurs from the date of expiry of the relevant period in relation to the first Part 36 offer, and not the date on which the improved Part 36 offer was made. The improved offer is not treated as a withdrawal of the first Part 36 offer, but the making of a new offer on improved terms. See CPR r.36.9(5). As there are two relevant offers to consider, and Asad has failed to obtain a judgment more advantageous than either of them, then RL is likely to be entitled to its costs from the date of expiry of the relevant period in relation to the first offer. See CPR r.36.17 (1) (a) and (3).
[B] Is not the best answer as it fails to take account of the fact that there are two Part 36 offers here and Asad failed to obtain a judgment more advantageous than either of them. See CPR r.36.17 (1) (a) and (3).
[C] Is the best answer as it takes full account of the fact that Asad won the action but failed to obtain a judgment more advantageous than the amount of either of the two Part 36 offers. It should therefore pay RL’s costs from the date of expiry of the relevant period in relation to the first offer (which is not withdrawn by the making of the improved offer – see CPR r. 36.9(5)), and interest on those costs. See CPR r.36.17 (1) (a) and (3).
[D] Is not the best answer as it fails to take account of the fact that Asad won the action and obtained judgment in the sum of £12,000. On the basis of the general principle that the overall winner is entitled to its costs, it is likely that the court would order that it was entitled to at least some of its costs. The most appropriate order would be that specified in CPR r.36.17 (3).

How well did you know this?
1
Not at all
2
3
4
5
Perfectly
3
Q

The court is most likely to
[A] allocate to the multi-track because the trial is likely to last more than one day.
[B] allocate to the fast track because the value of the claim is within the threshold for the fast track.
[C] allocate to the fast track because there is only one expert per party, which falls within the limitations for the fast track.
[D] allocate to the multi-track because the claimant’s wishes are determinative of the track where there is a dispute on allocation.

A

ANSWER:
[A] allocate to the multi-track because the trial is likely to last more than one day.
[A] is the best answer because the fast track is the normal track for the claims valued at between £10,000 and £25,000 only if the court considers that the trial is likely to last for no longer than one day: CPR 26.6(5), CPR 26.6(6). On the facts of this matter, the time estimate is clearly in excess of one day, which would put it in the scope of the multi-track, this is supported by the number of witnesses to be called, expert evidence and the court’s own provisional allocation. This is best reflected in answer A.
Although PD 26 para 9.1(3)(c) says that the possibility of trial lasting longer than a day is not a conclusive reason for allocation to the MT. Here the 1.5 days’ time limit is a certainty not a possibility and is an ‘at least‘ estimate so may be more.
On the facts of this matter, the time estimate is clearly in excess of one day, which would put it in the scope of the multi-track. This is best reflected in answer A.
[B] B is not the best answer because, as CPR 26.6(5) shows, the value alone is not determinative. B is incorrect because, as 26.6(5) shows, the value alone is not determinative.
[C] is not the best answer because, although it is correct that fast track trials restrict expert evidence to one expert per party, the length of trial estimate would still put the case on to the multi-track. CPR 26.6(5)
[D] is not the best answer as the views expressed by the parties are not determinative as allocation is a decision for the court. PD 26 para 7.5

How well did you know this?
1
Not at all
2
3
4
5
Perfectly
4
Q

Wayford should make an application to the court for an order seeking the information sought
[A] and need not serve a copy of the application on Landmark. The court may deal with the application without a hearing.
[B] but must wait a further seven days before doing so, as Landmark is entitled to a reasonable time to respond to the Request.
[C] and serve a copy of the application notice on Landmark at least 3 clear days before the hearing.
[D] supported by written evidence. Landmark is entitled to serve evidence in response.

A

ANSWER:
[A] and need not serve a copy of the application on Landmark. The court may deal with the application without a hearing.
[A] is the best advice. 18PD.5 paragraph 5.5(1) and (2) provides that where the second party has made no response to the Request, the first party need not, serve the application notice on the second party, and the court may deal with the application without a hearing. This only applies if at least 14 days have passed since the Request was served (as is the case here) and the time stating in it for a response has also expired (as is the case here).
[B] is incorrect as although Landmark is entitled to a “reasonable time to respond” to the Request (18PD 1 paragraph 1.1), as noted in A, 18PD.5 paragraph 5.5(2) indicates that as long as at least 14 days has passed since the Request (and the time stated in it for the response has expired) that this will be enough time. Landmark was served with the Request on 1 December, and we are told that it is now 15 December and so at least 14 days have passed and so there is no requirement to give Landmark a further 7 days to respond.
[C] is not the best advice (although it could be done) as whilst it is usual to serve an application notice as soon as practicable and at least 3 clear days before the hearing, for the reasons stated in A, this is not necessary on this fact pattern as Landmark has not responded within time.
[D] is not the best advice as it suggests that the application should be supported by written evidence, which is not necessarily the case. See 18PD.5 paragraph 5.4. Whilst both parties should consider whether evidence is required, it is not a requirement. 

How well did you know this?
1
Not at all
2
3
4
5
Perfectly
5
Q

Apart from the costs of the action, what additional order or direction must the court consider making?
[A] Attaching a penal notice to the order.
[B] Striking out the claim.
[C] Making a civil restraint order.
[D] Ordering the Claimant to make a payment into Court.

A

[C] Making a civil restraint order.

How well did you know this?
1
Not at all
2
3
4
5
Perfectly
6
Q

Which of the following is the correct advice to give Malcolm in relation to who should be parties to the proceedings?
[A] Malcolm should be named as the only Claimant, and Brian as the only Defendant.
[B] Malcolm and Derek must be named as Claimants, and Brian as Defendant.
[C] Malcolm should be named as Claimant, and Brian and Derek should be named as Defendants.
[D] Malcolm should be named as Claimant, Brian should be named as Defendant, and Derek should be named as an Additional Party.

A

ANSWER:
[C] Malcolm should be named as Claimant, and Brian and Derek should be named as Defendants.
[A] is an incorrect answer. Derek should also be named as Defendant as he has joint entitlement with Malcom CPR 19.3
[B] is an incorrect answer. Derek should be added as a Defendant not a Claimant. CPR 19.3
[C] Is correct. As Derek has a joint entitlement with Malcolm, he should be named as a Defendant: CPR 19.3
[D] is an incorrect answer. Derek should be added as a Defendant not an additional party. CPR 19.3

How well did you know this?
1
Not at all
2
3
4
5
Perfectly
7
Q

Where should Grace serve the proceedings on Henry?
[A] Henry’s temporary residential address.
[B] Henry’s usual residential address.
[C] Henry’s business address.
[D] Henry’s solicitors’ address.

A

ANSWER:
[D] Henry’s solicitors’ address.
[A] Wrong. Where a solicitor acting for the Defendant has notified the Claimant in writing that the solicitor is instructed to accept service, the claim form must be served at that solicitor’s business address. This is not that address. See CPR 6.7(1)
[B] Wrong. Where a solicitor acting for the Defendant has notified the Claimant in writing that the solicitor is instructed to accept service, the claim form must be served at that solicitor’s business address. This is not that address. See CPR 6.7(1)
[C] Wrong. Where a solicitor acting for the Defendant has notified the Claimant in writing that the solicitor is instructed to accept service, the claim form must be served at that solicitor’s business address. This is not that address. See CPR 6.7(1)
[D] Correct. Where a solicitor acting for the Defendant has notified the Claimant in writing that the solicitor is instructed to accept service, the claim form must be served at that solicitor’s business address. See CPR 6.7(1).

How well did you know this?
1
Not at all
2
3
4
5
Perfectly
8
Q

An important issue between the parties is what was discussed at a meeting attended by each party’s managing director (and nobody else) at which the contract was signed. The two managing directors have given very different accounts of what was discussed at that meeting.

This dispute is:
[A] unsuitable for mediation because any settlement agreement could not be enforced without the need to commence further court proceedings.
[B] unsuitable for mediation because Sampson’s claim is for a monetary sum only.
[C] suitable for mediation, particularly as it may enable the parties to preserve their trading relationship.
[D] unsuitable for mediation because it will be for the court to resolve the conflicting evidence of the two managing directors.

A

ANSWER:
[C] suitable for mediation, particularly as it may enable the parties to preserve their trading relationship.
[A] INCORRECT. Jackson ADR Handbook (Third Edition, 2021) at 2.61: “If enforcement might be an issue, ADR may be best used after issue of proceedings so the outcome can be recorded in a consent order that may be enforced more easily”. That applies here, as proceedings have already been commenced. This point is further supported by Jackson ADR Handbook (Third Edition, 2021) at 19.10: “A Tomlin Order is a form of consent order… in a Tomlin Order the court orders that further proceedings in the claim by stayed, except for the purpose of carrying out the terms of the compromise…the order will also provide for each party to have liberty to apply to the court if necessary to compel compliance with the scheduled terms” – that paragraph is within the syllabus but in Syllabus Area 22 (judgments, orders and enforcement) rather than Syllabus Area 1)
[B] INCORRECT. Jackson ADR Handbook (Third Edition, 2021) at 2.37: “In an ADR process, the parties can deal with any issues between them…even terms going outside the areas in dispute”. There is no reason to suggest that the fact that damages are claimed (monetary) makes the case less suitable for mediation than where other remedies are sought.
[C] CORRECT. There are many advantages to mediation (see Jackson ADR Handbook (Third Edition, 2021) at 2.30 to 2.40) and nothing that makes mediation unsuitable (the considerations at Jackson ADR Handbook (Third Edition, 2021) at 2.49 to 2.61 do not give rise to any issues on these facts). The fact that the parties can deal with issues beyond those at stake in the proceedings, and that Sampson wants to preserve the relationship with Jupiter, both point towards ADR such as mediation – see Jackson ADR Handbook (Third Edition, 2021) at 2.37 and 2.38.
[D] INCORRECT. There is nothing to suggest that conflicting evidence makes mediation unsuitable (this is not suggested at Jackson ADR Handbook (Third Edition, 2021) at 2.49 to 2.61). Jackson ADR Handbook (Third Edition, 2021) at 2.55: “Complex facts may be appropriately considered in court, especially if cross-examination may be important. However, there can be significant risks in taking a complex factual dispute to court, which may be controlled through ADR”.

How well did you know this?
1
Not at all
2
3
4
5
Perfectly
9
Q

Proceedings are at an early stage and Kim and Liu are keen to settle. No offers have yet been made by either side. You advise that £20,000 would be an appropriate sum to accept in settlement.
What is the best advice to give Kim and Liu in the circumstances?
[A] Make a written offer marked “without prejudice save as to costs” to settle the claim for £20,000.
[B] Make a Part 36 offer to settle the claim for £20,000.
[C] Make an open offer to settle the claim for £20,000.
[D] Make a written offer marked simply “without prejudice” to settle the claim for £20,000.

A

ANSWER:
[B] Make a Part 36 offer to settle the claim for £20,000.
[A] This is not entirely without merit and could be done but would not have the advantages as to costs and the 10% uplift that a Part 36 offer would have (CPR 36.17(4)). Moreover, the written offer is not compliant with CPR 36.5(b) as it should be made clear that the offer is made pursuant to Part 36.
[B] This is the best answer because, if the offer is not accepted and the judgment obtained by Kim and Liu is at least as advantageous at trial as the proposals contained in their part 36 offer, the automatic costs consequences of Part 36 will follow, and Kim and Liu will be entitled to a 10% uplift in their damages. See CPR 36.17(4) and WB 2023 commentary at 36.17.4
[C] This is not the best option: there would be no advantage (and potentially some disadvantage) in the court knowing at trial that Kim and Liu were prepared to accept substantially less than claimed, and this would not have the favourable costs consequences of a Part 36 offer.
[D] This would have no advantages compared to [A], which is already sub-optimal, and would have the disadvantage that the court could not refer to this when considering costs, so would be even less useful.

How well did you know this?
1
Not at all
2
3
4
5
Perfectly
9
Q

Which of the following statements about early neutral evaluation (ENE) is correct?
[A] ENE can take place in parallel with litigation but must take place outside and independently of the litigation process.
[B] All parties must agree to engage in the process before ENE can take place in a case.
[C] ENE usually takes place early on in a case, but it can only be utilised up to the start of the trial.
[D] The parties can agree between themselves to control the amount and form of information that is placed before the evaluator.

A

ANSWER:
[D] The parties can agree between themselves to control the amount and form of information that is placed before the evaluator.
[A] is wrong. Jackson ADR Handbook (3rd Edition, 2021) at 22.03 and 22.15, while ENE can take place outside of the litigation process, it can also happen within it by way of judicial evaluation.
[B] is wrong. Jackson ADR Handbook (3rd Edition, 2021) at 22.01 ENE can “be undertaken at the request of one party only in relation to their own case”.
[C] is wrong. Jackson ADR Handbook (3rd Edition ,2021) at 22.05 “ENE can also be used to settle disputes that arise during an assessment of costs after the main proceedings have been included (sic)”. N.B. The authors clearly intended the extract to end with the word concluded. In any event, as the paragraph makes clear that ENE can be used to settle disputes during an assessment of costs, it follows that it is incorrect that it cannot be used after the start of the trial.
[D] is correct. Jackson ADR Handbook (3rd Edition 2021) at 22.10, “The parties can control the amount and form of the information that is placed before the evaluator…”.

How well did you know this?
1
Not at all
2
3
4
5
Perfectly
10
Q

Patricia no longer works for Prestige and has refused to provide a witness statement. Given the importance of her evidence, Prestige wants to call her as a witness at trial. Prestige believes that despite her reluctance to give evidence, Patricia would be a favourable witness. Witness statements are due to be exchanged in 10 days’ time.
Which of the following is correct as to the step(s) to be taken by Prestige if it wants to be able to call Patricia to give oral evidence at trial?
Prestige should
[A] serve a witness summary on Amira, setting out the substance of Patricia’s evidence, at the same time as the latest date for serving witness statements.
[B] apply for permission to serve a witness summary on Amira. Amira must be given at least 3 days’ notice of the application. If permission is granted, Prestige should serve the witness summary on Amira.
[C] issue and serve a witness summons on Patricia as soon as practicable, and no later than 7 days before the date fixed for trial.
[D] apply without notice for permission to serve a witness summary on Amira. If permission is granted, Prestige should serve the witness summary by the latest date for serving witness statements.

A

ANSWER:
[D] apply without notice for permission to serve a witness summary on Amira. If permission is granted, Prestige should serve the witness summary by the latest date for serving witness statements.
[A] incorrect as Prestige requires permission to serve a witness summary CPR 32.9- (1) .
[B] no notice required for a permission application. CPR 32.9 (1) (b)
[C] Failing to serve a summary or statement means that the witness cannot be called to give oral evidence without permission CPR 32.10 thus issuing a witness summons is incorrect.
[D] Correct - Patricia has refused to provide a witness statement. Prestige wants to call her as a witness at trial believing that she would be a favourable witness. Exchange of witness statements has not yet taken place. CPR 32.9(1) provides that a party who is required to serve a witness statement for use at trial but who is unable to obtain one, as here, may apply, without notice, for permission to serve a witness summary instead. CPR32.9(4) provides that a witness summary must be served within the period in which a witness statement would have had to be served.

How well did you know this?
1
Not at all
2
3
4
5
Perfectly
11
Q

The claim form does not include a claim for an injunction. Work has temporarily ceased for seven days for the cement foundations to set. Farooq wishes to apply for an order to prevent Ameer from:
(i) carrying out building work on Saturdays, Sundays and after 6pm on weekdays; and
(ii) causing or permitting loose bricks and other debris to fall into Farooq’s garden.
What is the correct advice to give Farooq about whether he should (1) amend the claim form to include a claim for an injunction, and (2) give notice of the application for an interim injunction?
[A] He should amend the claim form to include a claim for an injunction and should apply for an interim injunction without notice.
[B] He should amend the claim form to include a claim for an injunction and should apply for an interim injunction with notice.
[C] He does not need to amend the claim form to include a claim for an injunction but should apply for an interim injunction with notice.
[D] He does not need to amend the claim form to include a claim for an injunction but should apply for an interim injunction without notice.

A

ANSWER:
[C] He does not need to amend the claim form to include a claim for an injunction but should apply for an interim injunction with notice.
[A] is wrong because there is no need to amend claim form before making the
application for an interim injunction. See CPR r.25.1(4) and also wrong because it states the application should be without notice (see CPR r.25.3(1)
[B] is wrong because although it correctly states the application should be made on notice, it is incorrect in stating the claim form needs to be amended to include a claim for an injunction.
[C] is correct. The claim form should include a claim for an injunction where a perpetual injunction is sought as a final order. That is not the case here. An interim injunction can be sought whether or not a claim for an injunction is made in the claim form. See CPR r.25.1 (4)
From the facts, it would appear that the matter is not urgent (we are told that work has ceased for 7 days). The application should therefore be made on notice to Ameer (with the application notice and evidence in support being served on him as soon as practicable after issue and in any event not less than 3 days before the hearing – see CPR r.23.7(1)).
An application should be made without notice only if it appears to the court that there is a good reason for not giving notice (see CPR r. 25.3(1)). There must be some urgency which means that there is literally no time to give notice before the injunction is required to prevent the threatened wrongful act (see WB 2023 commentary at 25.3.2), and that does not appear to be the case on these facts.
CPR r.25.1 (4)
[D] is wrong because it states the application should be made without notice.

How well did you know this?
1
Not at all
2
3
4
5
Perfectly
12
Q

You act for Nadeem, a director of BIL. Marika has brought a new claim against Nadeem, who was responsible for planning and supervising the excavation works.
Nadeem is angry at having to respond to Marika’s new claim, this time in his capacity as a director of BIL.
What is the best advice to give Nadeem about how he should respond to Marika’s new claim?
Nadeem should file an acknowledgment of service and make an application
[A] for summary judgment as Marika has no real prospect of succeeding on the claim.
[B] for security for costs as there is reason to believe that Marika will be unable to pay Nadeem’s costs if ordered to do so.
[C] for a civil restraint order to prevent Marika continuing with her claim unless she first obtains the court’s permission.
[D] to strike out Marika’s claim on the basis that it is an abuse of the court’s process.

A

ANSWER:
[D] to strike out Marika’s claim on the basis that it is an abuse of the court’s process.
[A] This is not the best advice: although an application could be made, there is no evidence on the facts that Marika has no real prospect of succeeding on the claim given the latest expert’s report. This would involve an unnecessary application being made when it would be better to apply to strike out for abuse of process.
[B] This is not the best advice: although an application could be made the conditions for the application are not evident. That Marika cannot afford to have the damage to her house repaired does not equate to satisfying the condition that she taken steps in relation to her assets to make it difficult to enforce an order (CPR 25.13(2)(g), SA 16).
[C] This is not the best advice. A civil restraint order can only be made after the court has decided to strike out a case - see CPR 3.4(6). While an order might be sought in conjunction with another application, the application should not be made in isolation.
[D] This is the best advice: although there is no cause of action estoppel, this is precisely the sort of case where the court would at least seriously consider striking out the claim as an abuse of process (See CPR 3.4(2)(b) and the Henderson v. Henderson rule set out in paragraph 3.4.5 of the White Book 2023). The court must adopt a “broad merits-based approach”. and applies the principle of res judicata.

How well did you know this?
1
Not at all
2
3
4
5
Perfectly
13
Q

Salim’s claim raises several contentious points of law relating to whether Janet owed him a duty of care at all material times.
You have been asked to advise Janet as to what method of alternative dispute resolution (ADR) would be most suitable on these facts.
What is the best advice to give Janet in response?
Janet should attempt to settle Salim’s claim
[A] by mediation, because the differences between the parties are such that it will need the expertise of a mediator to resolve.
[B] by referral to a conciliator, who can facilitate a negotiation between the parties.
[C] by judicial early neutral evaluation, so that a Judge can offer a provisional view on the legal issues involved in the case.
[D] by non-judicial early neutral evaluation, so that the parties can be offered advice as to how to resolve the dispute which is binding upon them in court.

A

ANSWER:
[C] by judicial early neutral evaluation, so that a Judge can offer a provisional view on the legal issues involved in the case.
[A] [A] and [B] are not the best answers, because these forms of ADR will not be as conducive as a judicial ENE to resolving the legal issue that is the cause of this dispute.
[B] [A] and [B] are not the best answers, because these forms of ADR will not be as conducive as a judicial ENE to resolving the legal issue that is the cause of this dispute.
[C] is the best answer, given that the major obstacle to settling this dispute out of court is the contentious nature of the legal issues involved, which a judicial ENE would be the form of ADR best suited to resolving out of those given in the options (Jackson ADR Handbook (Third Edition, 2021) at 22.15 and 22.06)
[D] is wrong, because a judicial ENE is not binding (Jackson ADR Handbook (Third Edition, 2021) at 22.15)

How well did you know this?
1
Not at all
2
3
4
5
Perfectly
14
Q

H&O has failed to respond. Shabnaz wishes to make an application for specific disclosure which you estimate can be dealt with in a one-hour hearing. The Bristol District Registry has telephone conferencing facilities available.
What is the correct advice to give Shabnaz about the procedure for making an application for specific disclosure in these circumstances?
[A] Unless the court orders otherwise, the matter will be dealt with by a telephone hearing and the application notice must be served as soon as practicable and at least 3 days before the date of the hearing.
[B] The matter is not one which may be dealt with by a telephone hearing and so the application notice, written evidence, and draft of order sought must be served on H&O as soon as practicable and, in any event, 3 days before the date of the hearing.
[C] Unless the court orders otherwise, the matter will be dealt with by a telephone hearing and the application notice must be served as soon as practicable and at least 5 days before the date of the hearing.
[D] As the matter will be dealt with by way of a telephone hearing, the application can be made without serving an application notice.

A

ANSWER:
[C] Unless the court orders otherwise, the matter will be dealt with by a telephone hearing and the application notice must be served as soon as practicable and at least 5 days before the date of the hearing.
[A] is wrong as it refers to the timing of 3 days before the hearing which is not correct for telephone hearings.
[B] is wrong as it states that the matter is not suitable for a telephone hearing (when it is listed as such – an interim application with a time estimate of “no more than one hour”).
[C] See PD 23A paragraph 6.1 and 6.2 which sets out that in a telephone conference enabled court, interim applications (and other named applications) with a time estimate of no more than an hour (as is the case here), will be conducted by telephone unless the court otherwise orders. The interim application for specific disclosure is relevant and applicable here as an interim application, and we are told that the estimate of the time of the hearing also comes within that set out at PD 23A paragraph 6.2. In terms of the time for service of the application notice, this is 5 days (and not the usual 3 days) when we are dealing with a telephone hearing PD 23A paragraph 4.1A).
[D] is wrong because, as stated above, the notice needs to be given as soon as practicable and at least 5 days before the date of the hearing. There is nothing on the facts to suggest that there is any basis for this application being made without notice; and if it were, then it could not be made by way of telephone application (PD 23A paragraph 6.3(a)) although this is not on syllabus).

How well did you know this?
1
Not at all
2
3
4
5
Perfectly
15
Q

At trial, the Judge found for CodeBright in relation to part of the claim resulting in CodeBright being awarded £31,000, and the Judge found in favour of GadgIT in relation to its counterclaim in the sum of £20,000.
What is the correct advice to give CodeBright about payment and costs where the court gives a judgment of this type?
[A] As there is a balance in favour of CodeBright, it may order GadgIT to pay the balance and may make a separate order for costs against each party.
[B] Notwithstanding that there is a balance in favour of CodeBright, the court must order that GadgIT pay the sum of £31,000 to CodeBright and that CodeBright pay the sum of £20,000 to GadgIT, and must make a separate order for costs against each party.
[C] The court must make a sequential order for payment of the judgment sum owed to CodeBright within 14 days and the payment of the judgment sum owed to GadgIT within 14 days thereafter, and may make a separate order for costs against each party.
[D] As there is a balance in favour of GadgIT, it may order CodeBright to pay the balance and may make a separate order for costs against each party.

A

ANSWER:
[A] As there is a balance in favour of CodeBright, it may order GadgIT to pay the balance and may make a separate order for costs against each party.
[A] is correct. See CPR 40.13. This rule provides that where the court gives judgment for specified amounts both for the claimant on his claim and against the claimant on a counterclaim, if there is a balance in favour of one of the parties, it may order the party whose judgment is for the lesser amount to pay the balance. In addition, if this rule applies, the court may make a separate order for costs against each party. This rule applies here as the court has given judgment for CodeBright in relation to its claim in the sum of £31,000 (£46,500 - £15,500) and against CodeBright in relation to the counterclaim in the sum of £20,000.
[B] is incorrect. Under CPR 40.13, the court MAY order the party whose judgment is for the lesser amount to pay the balance, so it is open to the court to give two judgments, one for the claimant on its claim and the other for the defendant on its counterclaim. However, it would be incorrect to say that it MUST do this.
[C] is incorrect. There is no obligation to make sequential order for payment. although it is generally correct to judgment sum should be paid within 14 days unless the court says otherwise (CPR 40.11), this is not relevant here. See CPR 40.13.
[D] is incorrect. There is no balance in favour of GadgIT; there is a balance in favour of CodeBright as the court has given judgment for CodeBright in relation to its claim in the sum of £31,000 (£46,500 - £15,500) and against CodeBright in relation to the counterclaim in the sum of £20,000, leaving a balance of £11,000 in favour of CodeBright.

How well did you know this?
1
Not at all
2
3
4
5
Perfectly
16
Q

Buckley is of the view that Star has no basis on which to defend the claim and is not responding in order to delay payment. In the circumstances, Buckley is keen to obtain judgment as soon as possible.
What is the correct advice to give Buckley as to whether it should take steps to obtain judgment in default?
[A] Buckley should request judgment in default today, because the time for filing an acknowledgment of service or defence has expired and the failure to include a response pack will not provide any basis for setting the judgment aside.
[B] Buckley should not request judgment in default today, because any judgment in default entered is likely to be set aside as having been wrongly entered.
[C] Buckley should request judgment in default today, but should be aware that Star may succeed in having the judgment set aside based on the failure to include a response pack.
[D] Buckley should not apply for judgment in default because the time for filing a defence has not yet expired. Instead, Buckley should make an application for summary judgment today based on the weakness of Star’s case.

A

ANSWER:
[C] Buckley should request judgment in default today, but should be aware that Star may succeed in having the judgment set aside based on the failure to include a response pack.
[A] is incorrect for the reason given below.
[B] is incorrect for the reason given below.
[C] is the correct answer, because it recognises that judgment in default can be obtained in these circumstances pursuant to CPR 12.3. However, this answer also identifies the risk that Star could seek to have any judgment set aside because of the failure to include a response pack. Such a failure may amount to “some other good reason” for a judgment in default to be set aside under CPR r13.3(1)(b) (White Book 2023 commentary at 13.3.2)
[D] is not the correct answer, because a claimant cannot apply for summary judgment without the court’s permission unless the defendant has filed either an acknowledgement of service or a defence (CPR r24.4)

How well did you know this?
1
Not at all
2
3
4
5
Perfectly
17
Q

The Claimant, who is represented by solicitors, has brought a claim against the Defendant for £45,000. The Defendant is a litigant in person. The matter has been allocated to the multi-track and the court has made no specific direction in relation to costs budgets. The Claimant has filed and served a costs budget with her Directions Questionnaire, but the Defendant has not done so.
What are the costs consequences, if any, of the Defendant’s failure to file a costs budget?
Unless the court otherwise orders,
[A] the Defendant will be treated as having filed a budget comprising only the applicable court fees.
[B] there will be no costs consequences.
[C] the Defendant must file an explanation of his failure to provide a costs budget when he files the agreed budget discussion report.
[D] the Defendant will not be able to recover his costs of the proceedings.

A

ANSWER:
[B] there will be no costs consequences.
[A] is not correct because although those are the correct consequences (CPR 3.14), they do not apply to the Defendant (as he is a litigant in person) CPR 3.13(1).
[B] is the correct answer. Because the Defendant is a litigant in person, he is not required to file a costs budget with his Directions Questionnaire, unless the court orders otherwise. Here, we are told that the court has made no specific directions about costs budgets and so the proviso does not apply. CPR 3.13(1).
[C] is not correct. CPR 3.13(1)
[D] is not correct. CPR 3.13(1)

How well did you know this?
1
Not at all
2
3
4
5
Perfectly
18
Q

You are instructed by Suri, the Claimant, in a breach of contract claim against Rebuildr Ltd (“RL”) relating to renovations at her home address. The quality of the work has been unsatisfactory, and the work has taken too long to complete. Following the letter of claim, a response has been received from RL which simply denies that the quality of work was poor and states that no complaints had been made whilst the works were ongoing. RL proposes a round-table meeting in an attempt to settle the dispute.
What is the correct action for Suri to take upon receipt of RL’s response?
[A] Suri should send a letter to RL in reply, requiring payment within 14 days and informing RL that, if payment is not received, proceedings will be issued.
[B] Suri should issue proceedings, as no further notice is required.
[C] Suri should consider the possibility of reaching a settlement, including making a Part 36 offer, and should continue to do so.
[D] Suri must agree to the round-table meeting as litigation should be a last resort.

A

ANSWER:
[C] Suri should consider the possibility of reaching a settlement, including making a Part 36 offer, and should continue to do so.
[A] is not correct because it would be contrary to the Practice-Direction-Pre-Action Conduct and Protocols
[B] is not correct because it would be contrary to the Practice-Direction-Pre-Action Conduct and Protocols as there are other requirements which should be observed e.g. canvassing options for settlement and a stocktake.
[C] is correct. PDPACAP C1-006 WB 2023, paras 8 and 9.
[D] is not correct because there is no requirement to have a meeting (although it might be a sensible thing to do and should not prevent considering the possibility of reaching a settlement).

How well did you know this?
1
Not at all
2
3
4
5
Perfectly
19
Q

Richard was visiting Judy’s shop when he was injured having tripped over a wire left trailing across the floor of Judy’s shop. Richard has issued a claim against Judy for the injury to his knee which he sustained.
Richard obtained a medical report from Dr Brown relating to his knee injury. Dr Brown’s medical report was attached to a covering email from Dr Brown sent to Richard’s solicitors. The email from Dr Brown stated that Richard was not engaging with his physiotherapy and that his recovery would be quicker if he were more engaged. The email from Dr Brown also stated that Judy’s insurer has been very slow in arranging follow-up physiotherapy sessions.
Richard’s solicitor inadvertently sent Dr Brown’s covering email to Judy’s solicitor together with Dr Brown’s medical report.
Will Judy be able to rely on Dr Brown’s covering email at the trial?
[A] Judy will not be able to rely on it in any circumstances because litigation privilege attaches to the email.
[B] Judy will be able to rely on it because litigation privilege has been waived with the inspection of the document.
[C] Judy will be able to rely on it if the court grants her permission to do so.
[D] Judy will not be able to rely on it because it did not form part of the medical report.

A

ANSWER:
[C] Judy will be able to rely on it if the court grants her permission to do so.
[A] Incorrect. Although the document was privileged and it was disclosed accidentally, CPR 31.20 permits reliance upon the document if the court gives permission.
[B] Incorrect. The court’s permission is still required for her to rely on it.
[C] Correct. CPR 31.20 permits reliance upon a document which has been inadvertently inspected if the court grants permission.
[D] Incorrect. There is no such provision. See explanation at C and CPR31.20. Judy may be able to rely upon the contents but only if the court gives permission.

How well did you know this?
1
Not at all
2
3
4
5
Perfectly
20
Q

Karen was employed by Brown Ink Ltd (“BI”). In September 2022, Karen was injured when she tripped and fell down a flight of stairs at work. In her subsequent letter of claim to BI, Karen asserted that the accident had been caused by a failure to provide a handrail and this amounted to negligence. The letter gave details of her injuries and indicated that she would be seeking evidence from a medical expert. The relevant Pre-Action Protocol was followed, but no settlement could be reached.
In December 2023, Karen issued and served proceedings against BI. Karen attached to her particulars of claim a medical report and a detailed schedule of past and future expenses and losses. BI disputes the claim, asserting that the absence of a handrail on the stairs did not cause the accident, but that it was Karen’s inappropriate footwear and lack of attention on the stairs that led to her fall. BI also denies the amount that Karen is claiming and asserts that BI’s medical report outlines that her injuries have been exaggerated.
What is the best advice to give to BI as to how it should respond to the claim?
[A] BI should state in its defence which matters in the Claimant’s medical report and items in the schedule of past and future expenses and losses it disputes (with reasons and alternative figures where appropriate) and attach its own medical report and counter-schedule of past and future expenses and losses to its defence.
[B] BI should state in its defence which matters in the Claimant’s medical report and items in the schedule of past and future expenses and losses it agrees with, disputes, or neither admits nor disputes but has no knowledge of, and attach its own medical report and counter-schedule of past and future expenses and losses.
[C] BI should state in its defence which matters in the Claimant’s medical report and items in the schedule of past and future expenses and losses it agrees with, disputes, and which it has no knowledge of, but need not serve its own medical report or counter-schedule of past and future expenses and losses until directions have been given.
[D] There is no need for BI to respond in its defence in detail other than to deny those matters contained in the Claimant’s medical report of which it has no knowledge; BI need not respond at all to the schedule of past and future expenses and losses which are for the Claimant to prove.

A

ANSWER:
[B] BI should state in its defence which matters in the Claimant’s medical report and items in the schedule of past and future expenses and losses it agrees with, disputes, or neither admits nor disputes but has no knowledge of, and attach its own medical report and counter-schedule of past and future expenses and losses.
[A] This is not incorrect but is NOT the best advice as CPR PD 16 para 11 states that the Defendant should also set out what he agrees with and neither agrees nor disputes but has no knowledge of. This would allow the issues to be narrowed in accordance with the overriding objective.
[B] is the CORRECT answer as it is the BEST ADVICE. See CPR PD 16 para 11.
[C] This is NOT the best advice as CPR PD 16 para 11 states that the Defendant should attach these to, or include in, his defence if he intends to rely on them. It would be preferable to do this at these earlier statements of case stage, rather than wait until disclosure or expert evidence is exchanged.
[D] This is NOT the best advice because, whilst the Claimant does have to prove all aspects of his case, CPR PD 16 para 11 provides for the Defendant to deal with these matters in his defence (or to be attached to it) so that issues can be narrowed at an early stage.

21
Q

In explaining his non-compliance in respect of Thiago’s statement, Dave referred to the fact that Thiago’s wife had been very ill and hospitalised before the deadline for exchange, so Thiago had been spending much of his time at the hospital or caring for his wife.
In respect of Dave’s application, what is the best advice to give him as to (1) the decision the court is most likely to make, and (2) its likely approach to that decision?
Dave’s application will
[A] succeed because the failure to comply with the unless order was not serious or significant, and the explanation given by Dave was a good reason for the default. To decide the application, the court will also consider in detail all the circumstances of the case, so as to enable it to deal with the application justly.
[B] succeed because, although the failure to comply with the unless order was serious or significant, the explanation given by Dave for the default was outside his control and was a good reason for the breach, allowing the court to grant relief from the sanction. To decide the application, the court will also consider all the circumstances of the case, so as to enable it to deal with the application justly.
[C] succeed because his failure to comply with the unless order was not serious or significant, allowing the court to grant relief from the sanction, without having to consider the reason for the breach or all of the circumstances of the case.
[D] fail because his failure to comply with the unless order was serious or significant, notwithstanding that the explanation given by Dave was a good reason for the default. To decide the application, the court will consider all of the circumstances of the case so as to enable it to deal with the application justly.

A

ANSWER:
[B] succeed because although the failure to comply with the unless order was serious or significant the explanation given by Dave for the default was outside his control and was a good reason for the breach, allowing the court to grant relief from the sanction. To decide the application, the court will also consider all the circumstances of the case, so as to enable it to deal with the application justly.
[A] This is possible, but does not best reflect the court’s likely reasoning. the breach would be likely to be found to be serious or significant (as an unless order has been breached; (although not necessarily as per 4th para at WB 2023 commentary at 3.9.4) Dave’s reason was also likely to be found to be a good one for the reasons set out in the answer to [B] above. However, it is likely that the court will have briefly considered all of the circumstances of the case rather than in detail, as per WB 2023 commentary at 3.9.5
[B] This best reflects the court’s reasoning. Paragraph 3.9.4 of the WB 2023 commentary (regarding the first stage of the Denton test) confirms that the very fact that a party has failed to comply with an unless order is undoubtedly a pointer towards seriousness and significance (British Gas Trading Ltd v Oak Cash and Carry Ltd [2016] EWCA Civ 153). However not every breach of an unless order is serious or significant. The court would then look at the second stage and consider why the default occurred. If some good reason is shown for the failure to comply with an order, the court will usually grant relief from any sanction imposed because of it.
Here, Dave’s key witness had been pre-occupied with caring for and visiting his very ill wife in hospital; paragraph 3.9.5 of the WB 2023 commentary gives this as an example of a good reason for default where the court is likely to grant relief (Cranford Community College v Cranford College Ltd [2014] EWHC 349 (IPEC)) and states that “good reasons are likely to arise from circumstances outside the control of the party at fault”. Rule 3.9(1) requires that, in every case, the court will consider ”all the circumstances of the case” (WB 2023 commentary at 3.9.6 - first para), however such consideration is likely to be brief (see second para WB 2023 commentary at 3.9.4 in relation to the first stage).
[C] This is possible, but does not best reflect the court’s reasoning. Paragraph 3.9.4 of the WB 2023 commentary (regarding the first stage of the Denton test) confirms that the very fact that where a party has failed to comply with an unless order it is undoubtedly a pointer towards seriousness and significance (British Gas Trading Ltd v Oak Cash and Carry Ltd [2016] EWCA Civ 153). Further, even if the breach was not serious or significant, the court would still be obliged to briefly consider the second stage and all the circumstances of the case. See CPR 3.9(1) and WB 2023 commentary at 3.9.6 and 2nd para of WB 2023 commentary at 3.9.4.
[D] This is possible, but does not best reflect the court’s reasoning. Paragraph 3.9.4 of the WB 2023 commentary (regarding the first stage of the Denton test) confirms that the very fact that where a party has failed to comply with an unless order it is undoubtedly a pointer towards seriousness and significance (British Gas Trading Ltd v Oak Cash and Carry Ltd [2016] EWCA Civ 153). Further, even if the breach was not serious or significant, the court would still be obliged to briefly consider the second stage and all the circumstances of the case. See CPR 3.9(1) and WB 2023 commentary at 3.9.6 and 2nd para of WB 2023 commentary at 3.9.4.

22
Q

At the meeting, the solicitors reached a verbal agreement that AL would buy back the shoes from Demetris at the same price as Demetris had paid AL. Unhappy with the outcome of the meeting, AL now contends that its solicitor did not have any authority to make this agreement and, therefore, AL is not bound by it.
Demetris asks for your advice as to whether AL is bound by the verbal agreement.
What is the best advice to give Demetris about whether the verbal agreement is binding on AL?
The verbal agreement is
[A] not binding on AL because it is not in writing and requires subsequent client approval.
[B] binding on AL because Demetris may rely on AL’s solicitor’s apparent authority to act on its behalf.
[C] not binding on AL because AL was not present at the meeting and cannot be bound by the detailed terms of the agreement reached unless made aware of them beforehand.
[D] not binding on AL because a negotiated agreement between solicitors is provisional only, and not binding without subsequent client approval.

A

ANSWER:
[B] binding on AL because Demetris may rely on AL’s solicitor’s apparent authority to act on its behalf.
[A] is not the best advice because it is incorrect and ignores the application of apparent or ostensible authority (Jackson ADR Handbook (3rd Edition, 2021) para 4.08). There is no requirement for the negotiation to be reduced to writing before it is binding and the agreement may be concluded orally and result in a binding contract.
[B] is the best advice: Jackson ADR Handbook (3rd Edition, 2021) para 4.08. that, in negotiating as an agent, as here, the solicitor will normally have apparent authority to settle on behalf of the client who they are representing so any agreement reached may be treated by the other side as binding. In any event, the facts suggest that AL knew of the purpose and the likely outcome of the settlement meeting i.e. settlement. Therefore, AL is bound by the agreement reached. The fact pattern is based to a degree on Waugh v MB Clifford cited in that paragraph.
[C] is not the best advice because a solicitor will normally have apparent authority to settle a claim which is the subject of proceedings on behalf of the client (Jackson ADR Handbook (3rd Edition, 2021) para 4.08) and, as here, the solicitor will be acting as the client’s agent and has apparent authority to negotiate on its behalf especially given that Al had knowledge of the purpose and likely outcome of the meeting.
[D] is not the best advice as a solicitor may bind their client as, in negotiating an agent, as here, the solicitor will normally have apparent authority to settle on behalf of the client. While it is possible for there to be a limit on the solicitor’s authority to negotiate, none is mentioned here and, in any event, should be expressed at the outset of the discussion as any agreement reached should be within the parameters of the client’s instructions. (Jackson ADR Handbook (3rd Edition, 2021) para 4.08).

23
Q

Lori brings a claim in the County Court against Cornelia for a debt owed under a contract. Cornelia defends the claim, and it is allocated to the fast track and heard by a District Judge. At trial, Lori succeeds.
Cornelia appeals. Her appeal is allowed by the Circuit Judge at the County Court.
Lori now wishes to appeal the Circuit Judge’s decision.
To which Court should she appeal, and what test will be applied when considering her application for permission to appeal?
Lori should appeal to the
[A] High Court, which will grant permission if her appeal has a real prospect of success or there is some other compelling reason for the appeal to be heard.
[B] Court of Appeal, which will grant permission if her appeal has a real prospect of success and there is some other compelling reason for the appeal to be heard.
[C] High Court, which will grant permission if her appeal has a real prospect of success; and raises an important point of principle or practice; or there is some other compelling reason for it to be heard.
[D] Court of Appeal, which will grant permission if her appeal has a real prospect of success; and raises an important point of principle or practice; or there is some other compelling reason for the Court of Appeal to hear it.

A

ANSWER:
[D] Court of Appeal, which will grant permission if her appeal has a real prospect of success; and raises an important point of principle or practice; or there is some other compelling reason for the Court of Appeal to hear it.
[A] This is not correct, as this is a second appeal, it must be made to the Court of Appeal: see PD52 A paragraph 4.7 . As a second appeal the test for permission is as stated at CPR 52.7(2).
[B] This is not correct, as this is not the correct test for a second appeal: see CPR R. 52.7(2). The appeal must have both a real prospect of success and raise an important point of principle or practice or there must be some other compelling reason for the Court of Appeal to hear it. This is not a conjunctive test.
[C] This is not correct, as this is a second appeal, it must be made to the Court of Appeal: see PD52 A paragraph 4.7.
[D] Correct - as this is a second appeal, it must be made to the Court of Appeal: see PD52A paragraph 4.7 . As a second appeal the test for permission is correctly stated - see CPR 52.7(2).

24
Q

As the parties were unable to reach agreement in the most recent review, Linda, an independent chartered surveyor, was selected by the parties. Both parties were satisfied with her suitability, expertise, and experience. Agreed instructions specified that Linda should set out calculations but was not otherwise required to give reasons for the assessment process or decision.
Baxters is happy with the review and determination, but Cityscape is not. Cityscape contends that Linda did not take into account enough comparable properties in her analysis and did not explain her reasons for the number of properties selected for comparison in her report. However, Linda did set out her calculations and Cityscape accepts all other aspects of the review and determination.
What is the correct advice to give Baxters in relation to the prospects of Cityscape challenging Linda’s expert determination through the court, if at all?
[A] Cityscape cannot apply to the court to challenge or set aside the determination because expert determination is not subject to the supervision of the court.
[B] The court is likely to set the determination aside because Linda did not explain the reasons for the number of comparative properties selected.
[C] The proceedings are unlikely to succeed as Linda has done what she was instructed to do whether or not her report contained reasons for the decision.
[D] The court is likely to set aside the determination and determine the rent review itself because Linda did not explain the reasons for the number of comparative properties selected.

A

ANSWER:
[C] The proceedings are unlikely to succeed as Linda has done what she was instructed to do whether or not her report contained reasons for the decision.
[A] wrong. It is possible to issue proceedings to challenge/set aside but they are unlikely to be successful. See para 24.05 Jackson ADR Handbook (Third Edition, 2021) as to situations where court may become involved (disputed jurisdiction or challenge of determination).
[B] Possible – see para 24.53 Jackson ADR Handbook (Third Edition, 2021) - but very unlikely on the available facts. The lease gives Linda final decision-making powers over both process and outcome. The instructions confirm no reasons are required in respect of assessment process or decision.
[C] See para 24.31 Jackson ADR Handbook (Third Edition, 2021).
The lease gives Linda final decision-making powers over both process and outcome. The instructions confirm no reasons are required in respect of assessment process or decision.
[D] Possible – see para 24.53 Jackson ADR Handbook (Third Edition, 2021) - but very unlikely on the available facts. The lease gives Linda final decision-making powers over both process and outcome. The instructions confirm no reasons are required in respect of assessment process or decision.

25
Q

Jennifer has also very recently discovered that Reed did not carry out a roof alteration in accordance with the contract specification. She wants to amend the particulars of claim to include this allegation.
What is the best advice to give Jennifer about whether she can make the proposed amendment?
[A] Jennifer can amend the particulars of claim without the court’s permission or Reed’s consent, but Reed may apply to the court for an order disallowing the amendment and Jennifer would then need to make a cross-application seeking permission to amend.
[B] Jennifer can amend the particulars of claim if she obtains the written consent of Reed or permission from the court.
[C] Jennifer can amend the particulars of claim with the court’s permission, which the court will only give if she can show that the new claim arises out of the same facts or substantially the same facts as the claim for which a remedy has already been claimed in the proceedings.
[D] Jennifer is unlikely to be able to amend the particulars of claim at this late stage as disclosure and the exchange of witness statements have taken place, and expert evidence is due to be exchanged shortly.

A

ANSWER:
[B] Jennifer can amend the particulars of claim if she obtains the written consent of Reed or permission from the court.
[A] Is not the best answer because it is incorrect, as written consent should be sought from Red or an application should be made to the court before the amendment is made. The second part of the statement is correct and sets out what is likely to happen if she simply went ahead and made the amendment without permission or consent. See CPR 17.1(2).
[B] is correct and the best answer here – see CPR r.17.1 (2).
[C] is not the best answer because it sets out the test for amending to add a new claim after the expiry of the limitation period (see CPR r.17.4(2)), which is not applicable on these facts (the breach was in 2020 so the original limitation period would not have expired until 2026).
[D] is not the best answer because, although the lateness of the amendment is something the court will consider, and a heavy onus lies on the party to justify the need for the amendment and explain the reason for the late amendment (see commentary in the WB 2023 at 17.3.8), the trial date has not yet been fixed, so the court is more likely to grant the amendment here, particularly as the roof defect appears to have been discovered recently.

26
Q

You represent Emma. Sam brings a claim against Emma for harassment in the County Court. Emma defends the claim. After a trial, the Judge decides to dismiss Sam’s claim on the grounds that (1) the claim was brought out of time, and (2) the conduct complained of did not amount to harassment (“the original decision”).
Sam obtains permission to appeal and files an appeal notice on the basis that the Judge was wrong on both grounds. Emma wishes to resist the appeal on the basis that the Judge was entirely correct on both grounds and she wishes the original decision to be upheld.
What is the correct advice to give Emma as to whether she needs to file a Respondent’s Notice, and why?
[A] Emma does not need to file a Respondent’s Notice as she only wishes the appeal court to uphold the original decision.
[B] Emma needs to file a Respondent’s Notice because she also needs permission to ask the appeal court to uphold the original decision.
[C] Emma needs to file a Respondent’s Notice because she is seeking to argue that the original decision was right on both grounds.
[D] Emma needs to file a Respondent’s Notice in order to be entitled to appear and make submissions at the hearing of the appeal.

A

ANSWER:
[A] Emma does not need to file a Respondent’s Notice as she only wishes the appeal court to uphold the original decision.
[A] This is the correct answer: see CPR R. 52.13(1) and (2). A respondent’s notice is not compulsory: it is required only when a respondent wishes to make a cross-appeal (CPR R. 52.13(2)(a)) or wants the appeal court to uphold the decision of the lower court for different reasons to those given by a lower court (CPR R. 52.13(2)(b)).
Here, Emma would not be seeking either: the fact pattern makes clear that she is asking the appeal court to uphold the Judge’s decision for the reasons that he/she made it on both grounds. Thus, no respondent’s notice is required.
[B] This is not the correct answer for the reasons given in [A].
[C] This is not the correct answer for the reasons given in [A].
[D] This is not the correct answer for the reasons given in [A]. Note in particular CPR R. 52.13(1): the respondent’s notice is optional.

27
Q

James is now ready to install the pipes on Robert’s land, but, on three separate occasions in the last four weeks, Robert has refused both James and his builder permission to come onto Robert’s land. James needs to have the water pipes installed before he can progress with the building of his house. James has now issued proceedings for breach of contract against Robert and has applied for an interim injunction which would require Robert to allow James onto Robert’s land to install the pipes.
You are instructed to represent James at the hearing of the interim injunction application.
Which of the following best describes the approach that the court will take when determining James’ application for an interim injunction?
[A] The court will consider whether it would be just and convenient to grant James’ application for an interim injunction.
[B] The court will apply the American Cyanamid guidelines when deciding whether to grant or withhold the injunction. This will include determining whether there is a serious question to be tried, the adequacy of damages to either party should it turn out that the injunction should not have been granted, and where the balance of convenience lies.
[C] The court will have to decide whether granting or withholding the injunction is more or less likely to cause irremediable prejudice if it turns out that the injunction should not have been granted or withheld. It will be legitimate for the court to consider whether it feels a high degree of assurance that James will be able to establish his right to an injunction at any future trial.
[D] The court will consider whether it would be fair and reasonable in all the circumstances to grant James’ application for an interim injunction.

A

ANSWER:
[C] The court will have to decide whether granting or withholding the injunction is more or less likely to cause irremediable prejudice if it turns out that the injunction should not have been granted or withheld. It will be legitimate for the court to consider whether it feels a high degree of assurance that James will be able to establish his right to an injunction at any future trial.
[A] is not the best answer. This answer correctly repeats the statutory wording of s. 37(1) of the Senior Courts Act 1981. But this section only creates the discretion for the court to grant an injunction, it does not adequately describe the approach that the court will take when it determines this application and for which see the explanation given at B.
[B] is not the best answer. This answer would be correct if James were applying for a prohibitory injunction. However, here he is applying for a mandatory injunction i.e. an injunction that will require Robert to permit James to enter onto his land. This is therefore not the approach that the court will take when determining the application.
[C] is the best answer. As set out in [B] above, this is an application for a mandatory injunction. Consequently, the approach to be adopted is that set out in Films Rover International Ltd v Cannon Film Sales Ltd [1987] 1 WLR 670 and extracted at para 15-24 of the WB Vol 2 2023. This answer most closely mirrors those principles.
[D] is not the best answer and is wrong. This is not part of any of the tests for the granting of either prohibitory or mandatory injunctions.

28
Q

Andrew has brought a claim for negligence against James. In accordance with the court’s directions, Andrew served a witness statement from Godfrey. At the trial, Andrew decides not to call Godfrey to give oral evidence and does not put in or rely on his statement as hearsay evidence.
James wishes to rely on Godfrey’s evidence. What is the correct advice to give James?
[A] James may not rely on Godfrey’s evidence.
[B] Upon application by James, the trial judge may direct Andrew to call Godfrey to give oral evidence.
[C] Upon application by James, the trial judge may permit James to call Godfrey to be cross-examined on the contents of his statement.
[D] James may put Godfrey’s witness statement in as hearsay evidence and does not need the court’s permission to do so.

A

ANSWER:
[D] James may put Godfrey’s witness statement in as hearsay evidence and does not need the court’s permission to do so.
[A]-[C] are wrong as James may rely upon the evidence and need not make an application – see CPR 32.5(5) [B] This is not correct, because it does not accurately reflect the rules on standard disclosure. See CPR r31.6 and the White Book 2022 commentary at 31.6.2 and 31.6.8
[D] CPR 32.5(5) provides that if a party who has served a witness statement does not call the witness to give evidence at trial; or put the witness statement in as hearsay evidence, any other party may put the witness statement in as hearsay evidence.

29
Q

In a case for breach of contract against Hari, the court has made an order for standard disclosure. Ruth, the Claimant, insists that she has the right to inspect Hari’s annotated copy of the relevant contract (‘the annotated copy contract’). Ruth has requested, in writing, the annotated copy contract which Hari did not disclose in his disclosure list. Hari does not dispute that he has the annotated copy contract, and he has referred to it in his witness statement, which has been served. Ruth makes an application for inspection of the annotated copy contract.
What approach is the court likely to take in relation to Ruth’s application for inspection of the annotated copy contract?
The court will
[A] only order inspection of the annotated copy contract if Ruth can prove that inspection is necessary for the fair disposal of her claim.
[B] only order inspection if Ruth can prove that Hari was at fault in failing to include the annotated copy contract on his disclosure list.
[C] order inspection as Hari has referred to the annotated copy contract in his witness statement.
[D] not order inspection, as Hari’s private alterations on the annotated copy contract are unlikely to adversely affect his case or support Ruth’s case, and inspection is not required by a relevant Practice Direction.

A

ANSWER:
[C] order inspection as Hari has referred to the annotated copy contract in his witness statement.
[A] is incorrect – see explanation for [C].
[B] is incorrect – see explanation for [C].
[C] is correct. CPR 31.14(1)(b), headed “Documents referred to in statements of case” gives a general right to inspection of documents referred to in a witness statement providing that a party may inspect such documents. For that reason, the other options are incorrect as there is no provision requiring Ruth to prove the facts at A and B for inspection to be given and D is incorrect as it is clear that inspection is permitted by CPR31.14.
[D] is incorrect – see explanation for [C].

30
Q

Which of the following statements best describes how the court will assess costs on the standard basis?
[A] The court will disallow costs that have been unreasonably incurred or are unreasonable in amount and will resolve any doubt as to whether costs were reasonably incurred or reasonable in amount in favour of the receiving party.
[B] The court will disallow costs that have been unreasonably incurred or are unreasonable in amount and will resolve any doubt as to whether costs were reasonably incurred or reasonable in amount in favour of the paying party.
[C] The court will only allow costs which are proportionate to the matters in issue and will resolve any doubt as to whether costs were reasonably and proportionately incurred or reasonable and proportionate in amount in favour of the receiving party.
[D] The court will only allow costs which are proportionate to the matters in issue and will resolve any doubt as to whether costs were reasonably and proportionately incurred or reasonable and proportionate in amount in favour of the paying party.

A

ANSWER:
[D] The court will only allow costs which are proportionate to the matters in issue and will resolve any doubt as to whether costs were reasonably and proportionately incurred or reasonable and proportionate in amount in favour of the paying party.
[A] is wrong – see CPR r.44.3 (1) and (3). This describes assessment on an indemnity basis.
[B] is not the best answer – it omits the proportionality requirement but recognises the basis of assessment referred to in PD44 paragraph 6.2 but it is not a complete description. See CPR r, 44.3 (2)
[C] is wrong – it correctly identifies the proportionality requirement but wrongly states that any doubts are resolved in favour of receiving party instead of the paying party. See CPR r.44.3 (2).
[D] is the best answer – see CPR r.44.3 (1) and (2).

31
Q

Clare brought proceedings against Dr Dante for medical negligence. Clare’s solicitors posted the claim form and particulars of claim to Dr Dante by first-class post. However, unbeknown to the parties, Dr Dante never received them.
After the expiry of the deadline for Dr Dante to file an acknowledgment of service, Clare immediately requested, and obtained, default judgment.
Upon being served with notice of the default judgment, Dr Dante instructs solicitors who telephone you for advice about how Dr Dante should now proceed with regard to the default judgment.
What is the correct advice to give Dr Dante’s solicitors about how he should now proceed?
[A] Dr Dante’s solicitors should write to Clare’s solicitors informing them that Dr Dante never received the claim form and that the judgment is, therefore, invalid. That being so, Clare’s solicitors should be asked to write to the court asking that the default judgment be set aside.
[B] Dr Dante should apply to the court promptly for judgment to be set aside; the court must set aside the default judgment where the claim form has not been received; the court has no discretion in this regard.
[C] Dr Dante should apply to the court promptly for judgment to be set aside; the court may set aside the judgment providing it is satisfied that Dr Dante has a real prospect of successfully defending the claim or it appears there is some other good reason why the judgment should be set aside or the Defendant should be allowed to defend the claim.
[D] Dr Dante should apply to the court promptly for judgment to be set aside; the court may set aside the judgment providing it is satisfied that Dr Dante has a real prospect of successfully defending the claim and it appears that there is some other good reason why the judgment should be set aside and some other good reason why the Defendant should be allowed to defend the claim.

A

ANSWER:
[C] Dr Dante should apply to the court promptly for judgment to be set aside; the court may set aside the judgment providing it is satisfied that Dr Dante has a real prospect of successfully defending the claim or it appears there is some other good reason why the judgment should be set aside or the Defendant should be allowed to defend the claim.
[A] is incorrect. Although this could be done, the advice is incorrect on the reasoning. The judgment is not invalid and it is not for the Claimant’s solicitors to ask for the judgment to be set aside. See reasoning at [C].
[B] is wrong. CPR 13.2 does not apply as the conditions in CPR 12.3(1) and 12.3(3) were satisfied.
[C] is correct. The documents were validly served by first class post under CPR 6.26 (SA 6) and are deemed served the second day after they were posted, whether or not they were in fact received. The conditions in CPR 12.3(1) and 12.3(3) were therefore satisfied. Therefore, Dr Dante cannot apply to set aside default judgment as of right under CPR 13.2. Dr Dante can, however, apply to set aside default judgment under CPR 13.3. Because he has only just discovered the situation, such an application is still “prompt” under CPR 13.3(2).
[D] is wrong. This wrongly states the grounds upon which the court may set aside judgment under CPR 13.3 and makes the test conjunctive.
CPR6.26, CPR12.3(1), CPR12.3(3), CPR 13.2 and CPR 13.3

32
Q

You are instructed by Mary, who sustained a fractured leg, whiplash, and chest injuries in a road traffic accident on 12 August 2022. Proceedings were issued in the County Court Money Claims Centre in Salford on 16 September 2023, but have not been served.
Mary wishes to rely on the evidence of a surgeon who has reported on the nature and extent of her injuries. She also wishes to claim for various personal items which were damaged beyond repair in the accident.
What advice should you give as to the documents which must be served, and the date by which they must be served, to ensure that service is properly effected under the CPR?
[A] The claim form only must be served no later than 16 January 2024.
[B] The claim form, particulars of claim and a schedule of special damages must be served no later than 16 December 2023. Medical evidence need not be served until after the first case management conference.
[C] The claim form, particulars of claim, a report from the surgeon and a schedule of special damages must be served no later than 16 December 2023.
[D] The claim form, particulars of claim, a report from the surgeon and a schedule of special damages must be served no later than 16 January 2024.

A

ANSWER:
[D] The claim form, particulars of claim, a report from the surgeon and a schedule of special damages must be served no later than 16 January 2024.
[A] Is not correct because CPR 7.4 (1) and (2) and 16PD 4, SA 8 states that P’s of C, medical evidence and a Schedule all have to be served within four months of the date of issue.
[B] is not correct - see [A] above.
[C] is not correct - see [A] above.
[D] CPR 7.4 (1) and (2) and 16PD 4, SA 8

33
Q

The court has made an order for standard disclosure in the case. Niklaus has in his possession a surveyor’s report (“the report”) which he obtained two years ago because he was concerned that the barn was becoming structurally unsound. The report concluded that the barn was not yet at risk of collapse but stated that if remedial work was not carried out, then the risk of collapse would increase in the next two to five years. The report also identified that if the barn collapsed there was a risk of collateral damage to the neighbouring property.
Niklaus sends the report to his solicitors who seek your advice as to whether they are required to disclose the report and permit Phillippe to inspect it, if he so chooses.
What is the correct advice to give Niklaus’ solicitors in relation to the disclosure and inspection of the report?
[A] The report is capable of adversely affecting Niklaus’ case and is not privileged; it therefore must be disclosed and made available to Phillippe for inspection if he asks to see it.
[B] The report attracts legal professional privilege; it therefore must not be disclosed to Phillippe.
[C] The report attracts legal professional privilege; it therefore must be disclosed to Phillippe but should not be made available for inspection.
[D] The report was obtained before litigation was commenced or contemplated; it therefore does not need to be disclosed or made available to Phillippe for inspection.

A

ANSWER:
[A] The report is capable of adversely affecting Niklaus’ case and is not privileged; it therefore must be disclosed and made available to Phillippe for inspection if he asks to see it.
[A] is the correct answer. Standard disclosure requires Niklaus to disclose documents which are capable of undermining his case. His defence is predicated on the lack of foreseeability of damage. The report makes clear that the damage was foreseeable and it therefore meets the test for disclosure under CPR 31.6(b). It is not privileged because it did not “come into existence for the purpose of obtaining legal advice in existing or anticipate proceedings.” WB 2023 Vol 1 commentary at 31.3.9. As a result, the report must be disclosed and made available for inspection.
[B] is wrong. The report is not privileged for the reason given in [A]. Even if it were, Niklaus would still be required to disclose its existence to Phillippe but would have the right to withhold inspection of it pursuant to CPR 31.3(1)(b).
[C] is wrong. The report is not privileged for the reason given in [A].
[D] is wrong. This answer is a misunderstanding of the rule on litigation privilege. The fact that a document was created prior to litigation commencing or being contemplated means that litigation privilege cannot attach to it. It does not mean that the document is not disclosable.

34
Q

You act for Glossop Limited (“Glossop”). Glossop is in dispute with Carruthers PLC (“Carruthers”) about a consignment of office furniture sold recently to it by Carruthers. Glossop alleges the furniture is not of satisfactory quality. The contract for the sale of the furniture contained a clause stating that any dispute arising out of the contract should be referred to arbitration. Glossop wishes to determine the dispute by arbitration but Carruthers disputes that the arbitration clause is binding.
On 22 November 2023, Glossop filed an arbitration claim form seeking to determine whether there was a valid arbitration agreement. The court issued the arbitration claim form on the same day.
Unless the court orders otherwise, what is the latest date by when the arbitration claim form must be served on Carruthers?
The arbitration claim form must be served on Carruthers no later than
[A] 21 December 2023.
[B] 21 March 2024.
[C] 22 December 2023.
[D] 22 March 2024.

A

ANSWER:
[C] 22 December 2023.
[A] Incorrect – see explanation for [C].
[B] Incorrect – see explanation for [C].
[C] is correct. See CPR 62.4(2) in Section 2E-11 Volume 2 of ‘Civil Procedure’ (the White Book 2023. When calculating periods of time, the day on which the period begins is not included (CPR 2.8)
The other options are wrong for the same reason that [C] is correct.
See CPR 62.4(2) in Section 2E-11 Volume 2 of ‘Civil Procedure’ (the White Book) 2023 (CPR 2.8)
[D] Incorrect – see explanation for [C].

35
Q

Jonny is a witness for the Defendant, Molar Solutions Limited (“MSL”), which has been sued for breach of contract in the County Court at Watford. The trial will be held the week after next. Since making his witness statement, Jonny has had a disagreement with MSL and has left their employment to work for a rival firm. His evidence remains very important for MSL. In the circumstances, MSL decide that a witness summons must be issued and served.
Which of the following is correct as to the process for serving the witness summons upon Jonny?
[A] In order to be binding the witness summons must be served at least 14 days before the date on which Jonny is required to attend before the court.
[B] At the time Jonny is served with the witness summons he must be offered a sum of money to compensate for his loss of time as a result of his attendance at trial.
[C] The witness summons must be served personally by MSL.
[D] Jonny must be served in person at his home address.

A

ANSWER:
[B] At the time Jonny is served with the witness summons he must be offered a sum of money to compensate for his loss of time as a result of his attendance at trial.
[A] is wrong - see CPR 34.5(1) – the correct time frame is 7 days before the date on which he is required to attend.
[B] is correct - see CPR 34.7(b)
[C] is wrong - see CPR 34.6
[D] is wrong as there is no such provision.

36
Q

Maddox Cooper, Jeffery Proctor, and Tilly Taylor traded until 31 December 2022 under the partnership name ‘Cherrytree Farm’. Maddox then left the partnership (“the business”) and Jeffery and Tilly continued to trade but under the new name ‘Cherry Farm Shop’.
Rupert wishes to pursue a breach of contract claim in relation to a supply of farm foods he received from the business in 2022. Rupert had dealt with Maddox in relation to the contract.
On the basis of the facts, what is the correct name for the proposed Defendant in the title of the proposed proceedings?
[A] Mr Maddox Cooper trading as Cherrytree Farm.
[B] Cherrytree Farm (A Firm).
[C] Cherry Farm Shop (A Firm).
[D] Mr Maddox Cooper of Cherrytree Farm.

A

ANSWER:
[B] Cherrytree Farm (A Firm).
[A] is wrong for the reasons set out above, on the basis of the facts that there is no reason for the claim to be brought directly against Maddox Cooper as an individual. Also, Maddox Cooper does not trade as Cherrytree Farm.
[B] PD 16 para 2.6(c)(i) and PD 7A Para 5A.3 - in the case of a partnership where parties are being sued in the name of the partnership the full name by which the partnership is known together with the words (‘A Firm’). There is nothing in the fact pattern to suggest that the partner, that is Maddox Cooper, is being sued as an individual. In short where a partnership has a name unless it is inappropriate to do so a claim must be brought in or against the name under which the partnership carried on business at the time the cause of action accrued.
[C] is wrong as at the time the cause of action accrued the partnership was trading under the name of Cherrytree Farm and not Cherry Farm Shop.
[D] is wrong as, as noted above, there is nothing on the facts to suggest that the claim is brought against Maddox as an individual and the correct way to refer to the correct name for the proposed defendant would be B for the reasons stated above

37
Q

On Wednesday 15 November 2023, a firm of solicitors hand-delivered a claim form to the court for issue. On Thursday 16 November 2023, the claim was issued, and 17th November 2023 was entered on the claim form as the date of issue. The claim form was returned to the solicitors to arrange service. On Thursday 23 November 2023, the claim form was placed in an envelope, properly addressed to the Defendant. A first-class stamp was placed on the envelope, which was then posted that day.
Which of the following correctly states when the proceedings are deemed to have been brought for the purposes of the Limitation Act 1980?
[A] Wednesday 15 November 2023
[B] Thursday 16 November 2023
[C] Thursday 23 November 2023
[D] Friday 24 November 2023

A

ANSWER:
[A] Wednesday 15 November 2023
[A] is correct. “…where the claim form as issued was received in the court office on a date earlier than the date on which it was issued by the court, the claim is ‘brought’ for the purposes of the Limitation Act 1980…on that earlier date”. WB 2023 Vol 2 commentary at 8-3.2.
[B] is incorrect. See the explanation in relation to [A] above.
[C] is incorrect. See the explanation in relation to [A] above.
[D] is incorrect. See the explanation in relation to [A] above.

38
Q

On 2 February 2023, Peter issued a claim against John for damages for breach of contract. On 7 September 2023, the judge entered judgment for £27,000. The order was sealed on 11 September 2023 and served on John on 15 September 2023.
Unless the court orders otherwise, from which date does interest run on the judgment?
[A] 2 February 2023
[B] 7 September 2023
[C] 11 September 2023
[D] 15 September 2023

A

ANSWER:
[B] 7 September 2023
[A] is wrong because interest runs from the date that judgment is given.
[B] is correct. Rule 40.8 (1) - interest runs from the date that judgment is given.
[C] is wrong because interest runs from the date that judgment is given.
[D] is wrong because interest runs from the date that judgment is given.

39
Q

PHI’s solicitors consider making an application against EPL under the Norwich Pharmacal jurisdiction, seeking call records for the mobile phone which they believe will show the location of the phone belonging to Barbara in the relevant period. They seek your advice as to the prospects of succeeding in that application.
You advise that such an application is unlikely to succeed.
For what reason is the application likely to fail?
The application is likely to fail because
[A] such an application can only be made in order to establish the identity of the ultimate wrongdoer and Barbara’s identity is already known.
[B] EPL is a mere witness to Barbara’s wrongdoing and there is no need for such an order to enable an action to be brought against Barbara.
[C] EPL must be shown to be the ultimate wrongdoer for the application to succeed.
[D] EPL must also be an intended party to the proceedings which PHI wishes to bring against Alan and Barbara.

A

ANSWER:
[B] EPL is a mere witness to Barbara’s wrongdoing and there is no need for such an order to enable an action to be brought against Barbara.
[A] is wrong because although applications are most commonly brought for the purpose of obtaining disclosure of the identity of a person, they can be made for the disclosure of documents and other information; WB 2023 commentary at 31.18.2.
[B] is correct. The requirements for relief are set out in Mitsui & Co Ltd v Nexen Petroleum UK Ltd [2005] EWHC 622See commentary at paragraph 31.18.3 WB 2023 Vol 2. A wrong must have been carried out by an ultimate wrongdoer; there must be a need for an order to bring an action against the wrong doer and the person against whom the order is sought must have facilitated the wrongdoing and be able to provide the necessary information. Relief cannot be granted against a mere witness - see commentary at paragraph 31.18.6 WB 2023 Vol 1. A claim for disclosure does not lie against a defendant who is not a wrongdoer, neither facilitated nor committed the wrongdoing. See Various Claimants v News Group Newspaper Limited (No.2) [2013] EWHC. It cannot be said that EPL have in any way facilitated Barbara’s wrongdoing in acting dishonestly to defraud PHI.
[C] is wrong because the application can succeed even where a person is innocently caught up in the wrongdoing of another so that they are more than a mere witness and can therefore be compelled to disclose the identity of the wrongdoer or to give disclosure of documents and other information relating to the wrongdoer. However, the person against whom the order is sought must be mixed up so as to have facilitated the wrongdoing, which cannot be said to be the position here; WB 2023 commentary at 31.18.2 and 31.18.3.
[D] ] is wrong as CPR 31.18 provides that the NP order is for disclosure against the person who is not a party to proceedings. The person against whom the order is sought must be able to provide information necessary to enable the ultimate wrongdoer to be sued; WB 2023 commentary at 31.18.3 and 31.18.2 i.e. that the person who is more than a mere witness can be compelled to disclose the identity of the wrongdoer so that proceedings may be brought against the proper defendant.
WB 2023 commentary at 31.18.2, 31.18.3 and 13.18.6.

40
Q

Sylvester (aged 17), an intelligent college student, has agreed a settlement of £4,500 in respect of his personal injury claim arising from distressing and potentially embarrassing circumstances and in which liability is denied. Proceedings have not been issued. The court’s approval of the settlement is to be sought.
Which of the following statements is correct with regard to the approval of the settlement of Sylvester’s claim?
[A] Proceedings must be begun under Part 7 as liability is denied.
[B] The approval of the settlement except at trial will normally be heard by a Circuit Judge.
[C] It is not necessary to produce an opinion on the merits of the settlement as the settlement is less than £5,000.
[D] The court may make an order permitting Sylvester to conduct proceedings without a litigation friend.

A

ANSWER:
[D] The court may make an order permitting Sylvester to conduct proceedings without a litigation friend.
[A] This is incorrect. The appropriate procedure is set out under CPR8 and even though liability is denied that is no bar to approval of a settlement reached between the parties although the court will expect to be advised about the extent to which the defendant admits liability PD21 para 5.1(2)
[B] is incorrect – PD 21 para 6.5 - the approval of the settlement except at trial will normally be heard by a District Judge or Master.
[C] This is wrong as PD21 para 5.2(1) says that an opinion on the merits of the settlement or compromise given by counsel or solicitor acting for the child or protected party must, except in very clear cases, be obtained. The facts say that the circumstances were unusual, and liability is denied i.e. not very clear and there is no provision regarding the value of the claim relative to the need for counsel’s opinion.
[D] is the correct answer. Although CPR 21.2(2) says that a child must have a litigation friend CPR 21.2(3) provides that the court may make an order permitting a child to conduct proceedings without a litigation friend. This is possible as Sylvester is 17 and so it is wrong in the particular circumstances to say that he must appoint a litigation friend.

41
Q

Where a witness of fact states an opinion on a matter on which that witness is not qualified to give evidence, in what circumstances (if any) is that evidence admissible?
[A] Where the statement of opinion relates to a matter in respect of which there is no recognised body of expert opinion.
[B] Where the statement of opinion is made as a way of conveying relevant facts personally perceived by the witness.
[C] Where the statement of opinion relates to what the witness believes a business person would understand a contract term to mean.
[D] There are no circumstances in which non-expert opinion evidence is admissible.

A

ANSWER:
[B] Where the statement of opinion is made as a way of conveying relevant facts personally perceived by the witness.
[A] is incorrect because if there is no body of expert opinion then that is strongly suggestive of any opinion evidence on the matter being irrelevant and inadmissible.
[B] is correct. This is established by section 3(2) Civil Evidence Act 1972, White Book 2023, Volume 2 para 9B-1060. (It is on syllabus – see second bullet point of material to be assessed on page 30).
[C] is incorrect because contractual construction is generally not an expert issue, or if it is then it is an expert issue such that the witness must be qualified to give the relevant opinion evidence.
[D] is incorrect because section 3(2) Civil Evidence Act 1972 provides for a relevant circumstance.
Section 3(2) Civil Evidence Act 1972, White Book 2023, Volume 2 para 9B-1060

42
Q

You act for Jericho Limited (“Jericho”). Jericho supplied temporary staff to Venture Limited (“Venture”) under a contract by which Venture agreed to pay Jericho for the temporary staff provided. Jericho has issued and served proceedings alleging that, in breach of contract, Venture has failed to pay for the staff which it used. Venture has filed a defence denying that Jericho supplied it with any temporary staff and claims to have used another agency to provide temporary workers.
Venture has today emailed Jericho offering, in open correspondence, to mediate in relation to this dispute. Jericho does not want to engage in a mediation until Venture has provided business records which Jericho believes would show whether or not Venture has in fact used the temporary staff which Jericho sent to Venture.
What is the best advice to give to Jericho as to how to respond to Venture’s offer to mediate?
[A] Write a letter “without prejudice save as to costs” refusing to engage in a mediation.
[B] Write a letter “without prejudice” agreeing to engage in a mediation if the business records are provided.
[C] Write an open letter agreeing to engage in a mediation.
[D] Write an open letter agreeing to engage in a mediation if the business records are provided.

A

ANSWER:
[D] Write an open letter agreeing to engage in a mediation if the business records are provided.
[A] is not the best advice. A refusal to mediate should be accompanied by reasons: Jackson ADR Handbook (Third Edition, 2021) at 11.63. In addition, responding on a ‘without prejudice save as to costs’ basis to an open email means that when the court comes to consider the party’s conduct in relation to mediation (pre-costs considerations, for example at a case management conference – see Jackson ADR Handbook (Third Edition, 2021) at 9.22-9.23; also see Jackson ADR Handbook (Third Edition, 2021) at 9.05 “Judges are prepared to question parties as to the steps taken to attempt settlement and avoid litigation and will robustly encourage them to cooperate in the use of ADR”), it will only have sight of venture’s offer, which could prejudice Jericho or could lead to the court staying the claim for ADR without considering the differing positions in relation to documents. See also the feedback in relation to answer [D] below].
[B] is not the best advice. Agreeing to mediate if the business records are provided is good advice but writing the letter ‘without prejudice’ rather than on an open basis is not the best advice – see the feedback in relation to [A] above on this point. A court may be prepared to order specific disclosure to support the use of ADR (Jackson ADR Handbook (Third Edition, 2021) at 2.53) but cannot do so if it is not made aware at the appropriate stage of the need for disclosure. See also the feedback in relation to answer [D] below.
[C] is not the best advice. Mediation has the best prospects of success when the parties have sufficient information to be able to sufficiently evaluate the case (see e.g. Jackson ADR Handbook (Third Edition, 2021) at 3.29), and it would be better to therefore indicate an agreement to mediate if the business records are provided. If Venture refuses to provide these records, then Jericho will be faced with a choice as to whether to refuse to mediate, or whether to mediate without the business records. At that stage, Jericho would need to be careful not to unreasonably refuse to mediate (Jackson ADR Handbook (Third Edition, 2021) at 11.07-08) although the need for further information before attempting ADR would be a relevant consideration (Jackson ADR Handbook (Third Edition, 2021) at 11.23). However, this dispute has not reached that stage yet, as it is not known whether Venture will object to providing the records before any mediation, and if so, why. For this reason, [D] is a better answer. See also the feedback in relation to answer [D] below.
[D] is the best advice. See generally Jackson ADR Handbook (Third Edition, 2021) at 11.63 - 11.64. A party refusing to mediate should give clear and full reasons. If lack of evidence / information is an obstacle to ADR, this must be canvassed with the party, and consideration given to providing the evidence / information during or before the ADR. For this reason, this is the best advice. Furthermore, writing in these terms is not refusing to mediate – it is agreeing to mediate and proposing terms. Venture’s position in relation to those terms is not yet known. See also the feedback for the distractors explaining why those distractors are less good.

43
Q

On 11 May 2020, Jim suffered personal injury after falling from a ladder while working on a construction site owned and managed by his employer Mutale. Jim sustained back injuries and was unable to work for six months.
On 29 August 2021 you advised Jim in conference that there was a strong case that the accident was due to Mutale’s failures to operate a safe system of work. Jim’s son, Scott, was also at the conference.
On 28 September 2021, Jim emailed you to say that he was considering whether to proceed with a claim and would be in touch later in the year.
On 30 December 2021, Jim died in circumstances wholly unrelated to his injuries. As sole executor, Scott is the personal representative of Jim’s estate and received the grant of probate on 23 March 2022.
It is now 5 December 2023. Scott wants to bring the claim for the benefit of Jim’s estate but is worried that it is too late to do so.
What is the correct advice to give Scott in relation to the limitation position?
[A] The claim is not statute barred but must be issued on or before 29 August 2024.
[B] The claim is statute barred as the limitation period expired on 11 May 2023.
[C] The claim is not statute barred. The limitation period will not expire until 23 March 2025, three years after the grant of probate.
[D] The claim is not statute barred. The limitation period expires on 30 December 2024.

A

ANSWER:
[D] The claim is not statute barred. The limitation period expires on 30 December 2024.
[A] is incorrect – see explanation for [D].
[B] is incorrect – see explanation for [D].
[C] is incorrect – see explanation for [D].
[D] Section 11(4) of the Limitation Act 1980 states that the limitation period for a personal injury claim is typically “three years from – (a) the date on which the cause of action accrued; or (b) the date of knowledge (if later) of the person injured”.
However, section 11(4) is subject to section 11(5), which states that where the injured died within that initial three year period, then the limitation period is “three years from – (a) the date of death; or (b) the date of the personal representative’s knowledge; whichever is the later”. Scott’s date of knowledge is 29 August 2021 but Jim’s death is later. Therefore the limitation period in this case runs from the date of Jim’s death (30 December 2021) and will therefore expire on 30 December 2024. Therefore D is the correct answer.

44
Q

You act for ID8 Designs (“ID8”), which has obtained judgment in the County Court against Abdul for £16,300 plus interest and costs in respect of architectural services provided. ID8 now seeks advice in relation to methods of enforcing the judgment against Abdul.
Abdul’s second home is owned in his sole name. His primary home, which has a mortgage secured against it, is registered in joint names with his wife. Abdul is a management consultant, employed by OWorks Limited, and has current and savings accounts with his local bank, both held in joint names with his wife. Abdul is also a keen sailor and owns a small sailing boat. When not sailing, he and his wife like to travel around the UK and Europe in their motorhome. Both of these assets are registered in his sole name.
Which of the following methods of enforcement is available to ID8 on these facts?
[A] A third-party debt order obtained in the County Court against the savings bank account held jointly by Abdul and his wife.
[B] A charging order obtained in the County Court as a charge on Abdul’s beneficial interest in the jointly owned primary home, subject to the mortgage.
[C] A warrant of control obtained in the County Court against the sailing boat and/or the motorhome.
[D] An attachment of earnings order obtained in the High Court as the judgment debt is for more than £5,000.

A

ANSWER:
[B] A charging order obtained in the County Court as a charge on Abdul’s beneficial interest in the jointly owned primary home, subject to the mortgage.
[A] is incorrect. While it is correct to say that an application for a third-party debt order may be made against a bank account (paragraph 72.0.1 of the WB 2023 editorial introduction) it is not correct to say that it can be obtained against a bank account in the joint names of the debtor and his wife. This is not possible and is in contrast to execution (paragraph 72.2.15 of the WB 2023 commentary).
[B] This is correct. See paragraphs 73.0.1 and 73.0.4 (2) of the WB 2023 editorial introduction.
[C] is incorrect. While it can be correct to use a warrant of control to execute against goods in the County Court (paragraph 83.0.3 of the WB 2023 editorial introduction) and that execution against goods could be used as a method of enforcement against the sailboat and/or motorhome, it is not correct for this method of enforcement to take place in the County Court. The second paragraph of the 83.0.17 WB 2023 editorial introduction makes it clear that by virtue of art. 8 of the High Court and County Court Jurisdiction Order 1991, a judgment or order of a county court for the payment of a sum of money which is sought to enforce wholly or partially by execution against goods “shall be enforced only in the county court where the sum which it is sought to enforce is less than £600”. The County Court judgment here is for more than £5,000 and it should therefore be transferred to the High Court for enforcement purposes, if ID8 is wanting to use execution against goods as the method of enforcement.
[D] is incorrect. It is not correct for the County Court judgment to be transferred to the High Court for enforcement where this method is intended to be used. Paragraph 89.0.1 of the WB 2023 editorial introduction makes it clear that the former power of the High Court to make attachment of earnings orders has been revoked (save for the power to make an order for secure payments under a High Court maintenance order, which is not applicable here) and that the majority of attachment of earnings orders are made in the County Court. A distractor is to mention the jurisdictional limits relevant to the High Court and County Court in relation to execution against goods as a method of enforcement (at paragraph 83.0.17 of the WB 2023 editorial introduction).

45
Q

You are instructed on behalf of Stacey in proceedings for breach of contract brought against Rajan. You attend the hearing of Stacey’s application to vary the trial window. The Judge orders that a variation of the trial window will be permitted and that the costs of the hearing are to be paid by Stacey in any event. At the end of the hearing, the Judge summarily assesses Rajan’s costs at £840 but makes no mention of when the costs are to be paid.
Within what period must Stacey pay Rajan’s costs?
[A] Within 7 days of receipt of the sealed court order.
[B] Within 14 days of receipt of the sealed court order.
[C] Within 7 days of judgment.
[D] Within 14 days of the hearing.

A

ANSWER:
[D] Within 14 days of the hearing.
[A] is wrong advice. The amount of costs has been decided by the Judge and the order takes effect from the day when it is made - CPR 40.7(1). Under CPR 44.7 (1) A party must comply with an order for the payment of costs within 14 days of (a) the date of the judgment or order if it states the amount of those costs. It is incorrect to wait for the sealed court order.
[B] is wrong - see [A].
[C] is wrong - an interim order has been made not a final judgment and in any event the same rules apply as to payment of costs following judgment - see [A].
[D] is the correct advice and follows both CPR 44.7 and CPR 40.

46
Q

Which of the following is a step the court may take where there has been a failure to comply with the Practice Direction – Pre-Action Conduct and Protocols?
[A] The court may give summary judgment for the Defendant on the basis of the Claimant’s failure to comply with the Practice Direction.
[B] The court may relieve the parties of the obligation to comply with the Practice Direction.
[C] The court may reduce any damages awarded to the Claimant to reflect the failure to comply with the Practice Direction.
[D] The court may require the parties to resolve the dispute by a specified method of alternative dispute resolution rather than allowing the claim to proceed.

A

ANSWER:
[B] The court may relieve the parties of the obligation to comply with the Practice Direction.
[A] is wrong. The court cannot give summary judgment on the basis of a failure to comply with the Practice Direction. The court might be willing to strike out a claim where there has been failure to comply under CPR 3.4(2)(c) but it would be wrong to use the procedure under CPR 24.2 as a sanction for failure to comply with a Practice Direction.
[B] is correct. Practice Direction – Pre-Action Conduct and Protocols para. 15(a) provides that where there has been non-compliance with the PD the court may relieve the parties of the obligation to comply or further comply with it.
[C] is wrong. Practice Direction – Pre-Action Conduct and Protocols para. 16(c) provides that where the claimant is at fault and has been awarded a sum of money then the court may make an order depriving that party of interest or award interest at a lower rate. However, the court does not have the power to reduce the sum awarded to punish non-compliance.
[D] is wrong. The court does not have the power to compel a party to use ADR. Jackson ADR Handbook (3rd Edition, 2021) at 1.22-23.

47
Q

Aryan (aged 5) suffered personal injuries when a piece of equipment fell on him while at FunZone, a soft play area. Proceedings were issued against FunZone by Deepa, Aryan’s mother, acting as his litigation friend. Following the exchange of witness statements, the parties negotiate and reach a settlement of the whole claim in the sum of £8,750.
What step(s) should Deepa now take with regard to the settlement?
Deepa should
[A] ask FunZone to pay the sum of £8,750 within 14 days of the settlement, as the settlement sum is less than £10,000, and, therefore, does not require approval.
[B] issue a Part 8 claim which must include a request for approval of the settlement.
[C] make an application to the court for approval of the settlement.
[D] ask FunZone to make a Part 36 offer in the sum of £8,750, which Deepa can then accept within the relevant period without requiring the court’s approval of the settlement.

A

ANSWER:
[C] make an application to the court for approval of the settlement.
[A] This is incorrect – see reasoning at [C]. There is no provision dispensing with approval of a settlement based on its value. Specifically, CPR 21.10 makes it clear that no settlement shall be valid without the approval of the court.
[B] This is incorrect. A Part 8 claim is appropriate where before proceedings in which a claim is made by or on behalf of a child are begun, an agreement is reached for the settlement of the claim and the sole purpose of proceedings is to obtain the approval of the claim (see CPR 21.10(2)). Here, Part 7 proceedings have already been issued, so Part 8 proceedings would not be necessary; an application to the court is what is required.
[C] This is correct. Where in any personal injury case a claim for damages for future pecuniary loss, by or on behalf of a child, is dealt with by agreement after proceedings have been issued, an application must be made for the court’s approval of the agreement (see CPR 21.10(1) and PD21 para 6.1).
[D] This is incorrect. See reasoning at [C]. Even if the sum were accepted as a Part 36 offer, such acceptance is not valid without the court’s approval A compromise made on behalf of a protected party by acceptance of a Part 36 offer requires the approval of the court under CPR 21.10 (and also CPR 36.11, SA 18).

48
Q

Your client, Preeti, is in a dispute with her business partner, Charlie, in relation to how the work that each of them has contributed to the partnership should be valued. The valuation of their work has an impact on the amount of money Preeti and Charlie are entitled to receive under the terms of the contract between the parties, namely their partnership agreement. Amongst other matters, the partnership agreement provides that valuation issues should be determined by a process of expert determination using a named accountant at an accountancy practice known to both parties.
Which rules, provisions or principles would govern the obligations of the accountant in determining this dispute in the first instance?
[A] The Civil Procedure Rules.
[B] The provisions of the Arbitration Act 1996.
[C] The provisions in Halsey.
[D] The provisions of the partnership agreement and any subsequent agreement between the parties and the accountant.

A

ANSWER:
[D] The provisions of the partnership agreement and any subsequent agreement between the parties and the accountant.
[A] is incorrect for the same reasons, and because expert determination is not subject to the supervision of the court – Jackson ADR Handbook (3rd Edition, 2021) at 24.05.
[B] is incorrect for the reasons that [D] is correct: there is nothing in the facts to suggest the parties have agreed to arbitration.
[C] is incorrect for the reasons that [D] is correct, and because the provisions in Halsey relate to unreasonable refusal to consider ADR and are not relevant to this situation. Jackson ADR Handbook (3rd Edition, 2021) at 11.07.
[D] Jackson ADR Handbook (3rd Edition, 2021) paragraphs 24.02-03: “the parties may contractually bind themselves, in advance of any dispute arising, to use expert determination to resolve a dispute…the relationship between the parties and the expert, the obligations of the expert, and the circumstances in which the decision will be final and binding on the parties will be primarily governed by the terms of the contract by which the expert is appointed”. See also Jackson ADR Handbook (3rd Edition, 2021) paragraph 24.10 – “a separate contract will also come into existence between the parties and the expert when the matter is referred to the appointed expert for determination”. Hence [D] is correct.

49
Q

You act for Cecil, who is employed by Beech Productions Limited (“BPL”). Cecil had an accident while at work, slipping on a temporary floor that had been installed at BPL.
Your instructing solicitors sent a letter of claim to BPL concerning the injuries Cecil sustained. BPL responded admitting liability but challenging quantum.
In accordance with the Pre-Action Protocol for Personal Injury Claims (“Pre-Action Protocol”), your instructing solicitors write to BPL’s insurers with the names of three proposed medical experts and provide their CVs. Your instructing solicitors receive no response from BPL within the 14 days specified for any objection, and so proceed to obtain a report from one of those experts, Dr Khan.
Dr Khan’s medical report is sent to BPL. However, BPL objects to Cecil relying on the report because, BPL states, the report was obtained without BPL’s agreement. BPL proposes obtaining a report from another expert, Dr Jones, who was not named as one of the three original experts, to be used in place of Dr Khan’s report.
Cecil asks your advice on whether to agree to a report from Dr Jones being obtained.
What is the correct advice to give to Cecil in response, and why?
Cecil should
[A] agree to BPL’s request to obtain a report from Dr Jones in place of Dr Khan’s report because BPL’s written agreement to using Dr Khan had not been obtained.
[B] agree to BPL obtaining and relying on the expert report of Dr Jones because BPL’s written agreement was not obtained to use Dr Khan as a nominated expert, but BPL should be informed that Cecil will continue to rely on Dr Khan’s report.
[C] not agree to BPL obtaining Dr Jones’ report because BPL was required by the Pre-Action Protocol to agree to the instruction of one of the nominated experts.
[D] not agree to BPL obtaining Dr Jones’ report because BPL did not formally object to Dr Khan or any of the nominated experts.

A

ANSWER:
[D] not agree to BPL obtaining Dr Jones’ report because BPL did not formally object to Dr Khan or any of the nominated experts.
[A] Not the best answer. Although he has ability to agree to the request under the Pre-Action Protocol for Personal Injury Claims at paragraph 7.8 there is no need to because the experts were proposed in accordance with paragraph 7.3 and no objection was received under paragraph 7.6. Express agreement is not required provided the opportunity to object is given. Under paragraph 7.8, in absence of an objection, BPL cannot rely upon their own report unless Cecil agrees, the court gives permission or the report has been amended (which it has not). Cecil should not agree so that BPL needs to apply to the court for permission to rely upon an alternative report.
[B] Not the best answer. Is a possible option but not the best one – see above explanation.
[C] Not the best answer. Although she can, and should, object there are no cost consequences stipulated under the Pre-Action Protocol for Personal Injury Claims at paragraph 7.8 for the Claimant where a report was obtained in absence of objections.
[D] The best answer as it means that BPL need to apply to the court for permission to rely upon the report and they are unlikely to be granted it in absence of a very good reason. See WB Vol 1 Pre-action Protocol C2-012 paragraphs 7.3,7.6 and 7.8 and explanation at [A]